Principal Ideal, Polynomial generators

Click For Summary
SUMMARY

The discussion centers on the properties of principal ideals in the polynomial ring R[x], where R is an integral domain. It establishes that for a principal ideal I, there exists a generator g(x) with minimal degree among all non-zero polynomials in I. The proof utilizes the division algorithm, demonstrating that if f(x) is in I, then the remainder r must equal zero when divided by g(x). Furthermore, it concludes that not every polynomial of minimal degree generates the ideal, as illustrated by the example of the ideal in Z, where 2x+4 is also of minimal degree but does not generate the same ideal.

PREREQUISITES
  • Understanding of integral domains and their properties
  • Familiarity with polynomial rings, specifically R[x]
  • Knowledge of the division algorithm in the context of polynomials
  • Concept of principal ideals and their generators
NEXT STEPS
  • Study the properties of integral domains and their implications on polynomial rings
  • Learn about the division algorithm for polynomials in R[x]
  • Explore examples of principal ideals in various integral domains
  • Investigate the conditions under which a polynomial of minimal degree generates an ideal
USEFUL FOR

Mathematics students, algebraists, and anyone studying abstract algebra, particularly those focusing on ring theory and polynomial ideals.

nugget
Messages
44
Reaction score
0

Homework Statement



Suppose R is an integral domain and I is a principal ideal in R[x], and I \neq {0}

a) Show I = <g(x)> for some g(x)\inR[x] that has minimal degree among all non-zero polynomials in I.

b) Is it necessarily true that I = <g(x)> for every g(x)\inR[x] that has minimal degree among all non-zero polynomials in I?

Homework Equations



Theorems, the division algorithm.

The Attempt at a Solution



For (a) we can take a g(x) of minimal degree as our generator; if deg(g) = 0, then g(x) is a unit in I and I = R[x], so we can assume deg(g)\geq1

Now we let f(x)\inI and apply the division algorithm.

f = qg + r, where q,r\inR[x] and deg(r)<deg(g)

This means that r = f - qg which is an element of I. By our choice of g having minimal degree, this means that r = 0. Otherwise r would be an element of I, and could be written as r = pg, p\inR[x]. (I'm not quite sure how best to explain the reasoning behind the fact that r must equal 0)

finishing the proof after this is fine.

b) I'm confused for this question, but I assume its a contradiction type answer, although I can't imagine a g(x) that wouldn't generate a principal ideal...

Please help!
 
Physics news on Phys.org
nugget said:
if deg(g) = 0, then g(x) is a unit in I

Why??

Now we let f(x)\inI and apply the division algorithm.

f = qg + r, where q,r\inR[x] and deg(r)<deg(g)

Did you prove that the division algorithm holds when R is simply an integral domain?

b) I'm confused for this question, but I assume its a contradiction type answer, although I can't imagine a g(x) that wouldn't generate a principal ideal...

Consider \mathbb{Z} and take the ideal generated by X+2. Then 2X+4 is in the ideal and is of minimal degree.
 
Hey,

regarding that deg(g)=0 bit, I guess I just assumed that a constant would be a unit in I. This would only be the case if R was a field, right?

Now I'm really confused for this question too; don't know how to prove the division algorithm, let alone for an integral domain.

I think I understand what you're saying for b)

Z is an integral domain so we can use that for R. b) is not true because <x+2> generates I and is of minimal degree, but <2x+4> can't generate I (it can't generate x+2, as Z doesn't contain fractions) but is also of minimal degree. Hence not every g(x) in R[x] of minimal degree generates I.
 
The trick for (a) is that you already know that I is principal. That is, you already know that I=<g(x)> for some g. The only thing you need to prove is that g is of minimal degree.
 
Question: A clock's minute hand has length 4 and its hour hand has length 3. What is the distance between the tips at the moment when it is increasing most rapidly?(Putnam Exam Question) Answer: Making assumption that both the hands moves at constant angular velocities, the answer is ## \sqrt{7} .## But don't you think this assumption is somewhat doubtful and wrong?

Similar threads

  • · Replies 1 ·
Replies
1
Views
2K
  • · Replies 13 ·
Replies
13
Views
3K
  • · Replies 5 ·
Replies
5
Views
2K
Replies
2
Views
1K
  • · Replies 1 ·
Replies
1
Views
2K
  • · Replies 13 ·
Replies
13
Views
2K
  • · Replies 24 ·
Replies
24
Views
4K
  • · Replies 11 ·
Replies
11
Views
2K
  • · Replies 12 ·
Replies
12
Views
2K
  • · Replies 1 ·
Replies
1
Views
1K